Blue Mixed Set Flashcards

1
Q

A small business owner decided to retire, so she offered her long-time employee a chance to buy the business for $1 million. She promised in writing to keep the offer open to him for 90 days and to give him enough time to secure financing once he accepted the offer. Over the next few days, the employee cashed out all his retirement accounts and took a second mortgage on his home to raise the funds to purchase the business. When he approached the business owner to discuss the details of the sale, she said that she changed her mind and was revoking her offer because she did not want to retire after all.
Was the owner’s revocation of her offer proper?

A
Yes, because it was an offer that could be revoked at will.

B
No, because the owner created an option contract by promising to keep the offer open for 90 days.

C
No, because the employee detrimentally relied on the offer.

D
No, because the offer constitutes a merchant’s firm offer.

A

A
Yes, because it was an offer that could be revoked at will.

The owner’s revocation of her offer was proper because the offer could be revoked at will. Generally, offers can be revoked at will by the offeror, even if she has promised not to revoke for a certain period of time. There are limitations on the offeror’s power to revoke, but none of those exceptions apply in this case. (B) is incorrect because an option contract requires that the offeree give consideration for the promise by the offeror to keep the offer open, and no consideration is indicated by the facts. (C) is also incorrect. Detrimental reliance can limit an offeror’s power to revoke where the offeror could reasonably expect that the offeree would rely to his detriment on the offer, and the offeree does so rely. However, this usually is limited to those situations in which the offeror would reasonably contemplate reliance by the offeree in using the offer before it is accepted; e.g., when a general contractor uses a subcontractor’s bid in making its own offer. Here, the offer itself included a promise by the owner to give the employee time to secure financing after the offer was accepted. Therefore, the owner had no reason to anticipate that the employee would take immediate steps to raise the purchase money before he even accepted the offer. (D) is incorrect because these facts are not an example of a merchant’s firm offer. A merchant’s firm offer does not apply to any offer by a merchant; it applies only to an offer under the UCC for the sale of goods where a signed writing gives assurances that the offer will be held open.

How well did you know this?
1
Not at all
2
3
4
5
Perfectly
2
Q

An art collector was interested in buying a painting from his neighbor. The neighbor told the collector that he could have the painting for $30,000. The collector wanted to think the purchase over. Therefore, the two agreed in writing that the neighbor would keep the offer open for 30 days in exchange for $500, which the collector paid. The terms of the written agreement provided that the offer would expire at 11:59 p.m. on September 30 if the collector failed to accept by that time. On September 20, the collector telephoned his neighbor and told him, “The more I think about it, the less I think that I want your painting.” The neighbor responded, “That’s your decision to make.” On September 26, one of the neighbor’s friends was visiting him, saw the painting, and offered his friend (the neighbor) $35,000 for it.
On September 27, the neighbor mailed a $50 check to the collector with a letter stating that he was terminating his offer to the collector regarding the painting and refunding 10% of the money that the collector paid him to keep the offer open. He mailed the letter at 11:59 p.m. on September 27. The collector received the letter at 11:30 a.m. on September 29. On September 28, at 9:30 a.m., the collector mailed a letter to his neighbor stating that he had decided to purchase the painting and a certified check in the amount of $30,000 was enclosed. Two hours later, the neighbor sold the painting to his friend for $35,000. The neighbor received the collector’s letter on October 1 and immediately mailed the check back to the collector.
Can the collector maintain a successful legal action against his neighbor?

A
Yes, because the neighbor sold the painting after the collector’s effective acceptance, and before the neighbor’s revocation became effective.

B
Yes, because in his revocation the neighbor did not refund the full $500 to the collector.

C
No, because the neighbor effectively revoked his offer before the collector accepted.

D
No, because the collector’s power to accept lapsed before he effectively accepted.

A

D
No, because the collector’s power to accept lapsed before he effectively accepted.

The collector’s power to accept lapsed because the option contract specified that the offer would expire at 11:59 p.m. on September 30. Hence, the power had to be exercised prior to that time and it was not. The mailbox rule does not apply to the exercise of options. In such cases, acceptance is effective when received by the offeror, here on October 1. Thus, (D) is correct. (A) is wrong because, for the reasons discussed above, the collector did not effectively accept before his option expired. (C) is wrong for two reasons: (i) a revocation is not effective until received; and (ii) because the contract is an option, the offeror’s power to terminate the offer through revocation is limited. Even if the revocation had arrived earlier, the neighbor lacked the power to revoke. (B) is irrelevant. Returning the consideration, in and of itself, would not give the offeror the power to revoke in an option situation.

How well did you know this?
1
Not at all
2
3
4
5
Perfectly
3
Q

Which of the following is not a factor in the test for the validity of government action under the Establishment Clause when no sect preference is involved?

A
The action has a secular purpose.

B
The action has a primary effect that neither advances nor inhibits religion.

C
The action does not produce excessive government entanglement with religion.

D
The government action is narrowly tailored to promote a compelling interest.

A

D. “The government action is narrowly tailored to promote a compelling interest” is not part of the test for determining the validity of government action under the Establishment Clause. This strict scrutiny test is applied to determine the validity of government action that includes a sect preference under the Establishment Clause.
The three-part Lemon test, applied in cases in which there is no preference for one religious group over another, is that:
1. The action has a secular purpose,
2. The action has a primary effect that neither advances nor inhibits religion, and
3. The action does not produce excessive government entanglement with religion.

How well did you know this?
1
Not at all
2
3
4
5
Perfectly
4
Q

Under current Supreme Court precedent, the First Amendment Free Exercise Clause __________.

A
prohibits government regulations that interfere with religious practices unless the government can prove that the regulation is necessary to achieve a compelling government interest

B
prohibits government from punishing conduct just because it is religious

C
prohibits laws of general applicability unless they have an exception for religiously motivated conduct

D
prohibits government regulations that interfere with religious practices unless the government can prove that the regulation is narrowly tailored to achieve an important government objective

A

B
prohibits government from punishing conduct just because it is religious

The Free Exercise Clause prohibits government from punishing conduct just because it is religious. If the intent of the law is to interfere with religion, or if the law punishes conduct solely because it is religious, the law is invalid. For example, a law may not prohibit ritual slaughter of chickens while otherwise allowing the slaughter of chickens.
The Free Exercise Clause does NOT prohibit laws of general applicability unless they have an exception for religiously motivated conduct. Religiously neutral laws of general applicability generally are valid under the Free Exercise Clause without religious exemptions with two historic exceptions: the Amish must be exempted from mandatory schooling beyond eighth grade, and workers fired for refusing to perform tasks on religious grounds may not automatically be exempted from unemployment compensation.
The Free Exercise Clause does NOT prohibit government regulations that interfere with religious practices unless the government can prove that the regulation is necessary to achieve a compelling government interest or that the regulation is narrowly tailored to achieve an important government objective. These choices reflect higher standards (strict scrutiny and intermediate scrutiny, respectively) than is currently used in Free Exercise cases. Older cases purportedly applied strict scrutiny, but the Supreme Court seemed often to have to stretch to find a compelling interest in such cases.

How well did you know this?
1
Not at all
2
3
4
5
Perfectly
5
Q

Which of the following statements is correct regarding government action challenged under the Due Process or Equal Protection Clause where no fundamental right or suspect or quasi-suspect classification is involved?

A
The law will be upheld unless it is arbitrary

B
The law must be the least burdensome means to achieve the legislative goal

C
The burden of proof is on the government to show that the law is necessary

D
The law is valid only if it is substantially related to a legitimate government purpose

A

A
The law will be upheld unless it is arbitrary

If government action is challenged under the Due Process or Equal Protection Clause, and no fundamental right or suspect or quasi-suspect classification is involved, the law will be upheld unless it is arbitrary or irrational. A rational basis standard applies.
The law is valid if it is rationally related to a legitimate government purpose; it need not be substantially related to a legitimate purpose.
The burden of proof is on the CHALLENGER to show that the law is unconstitutional, NOT on the government to show that it is necessary.
The law need NOT be the least burdensome means to achieve the legislative goal. The rational relationship test does not require a tight fit between the goal sought and the law employed-just a rational connection.

How well did you know this?
1
Not at all
2
3
4
5
Perfectly
6
Q

A state enacted health care legislation to provide comprehensive insurance coverage on prescription drugs for all of its citizens. The legislation provided state reimbursement for the cost of all prescription drugs with one exception-a drug commonly known as the “abortion pill,” which was prescribed to induce early term abortions without surgery. All other prescription drugs for pregnant women were covered. A pregnant woman who had received a prescription for the drug and was subsequently denied reimbursement filed suit in federal district court challenging the constitutionality of the legislation.
Which of the following best describes the appropriate standard by which the court should review the constitutionality of the state legislation?

A
Because the state legislation does not improperly discriminate against a suspect class or burden a fundamental right, the woman will have to show that the legislation is not rationally related to any legitimate state interest.

B
Because the state legislation discriminates against women by not providing coverage for all of their prescription medications as it does for men, the state will have to demonstrate that the legislation is substantially related to an important government interest.

C
Because the state legislation impinges on a woman’s constitutional right to choose whether to terminate her pregnancy, the state will have to show that the legislation does not constitute an undue burden on that right.

D
Because the state legislation discriminates against women seeking to exercise their fundamental right to terminate their pregnancy in favor of women incurring the regular expenses of pregnancy, the state will have to demonstrate that the legislation is necessary to vindicate a compelling state interest.

A
A
Because the state legislation does not improperly discriminate against a suspect class or burden a fundamental right, the woman will have to show that the legislation is not rationally related to any legitimate state interest.

The court should require the woman to show that the legislation is not rationally related to any legitimate state interest. The Supreme Court has held that the right of privacy includes the right of a woman to have an abortion under certain circumstances without undue interference from the government. However, neither federal nor state government is required to grant medical benefit payments for abortions, even if it grants benefits for childbirth services. The Court has held that a state’s failure to provide funding for a woman’s abortion decision does not constitute interference with her constitutional right to make that decision; hence, such legislation is valid unless the plaintiff can show that it is not rationally related to a legitimate state interest. [See Maher v. Roe (1977)] (B) is incorrect because the legislation does not create a gender-based classification that would require application of an intermediate scrutiny standard. The fact that the restriction applies to a drug prescribed only to women does not establish gender-based discrimination. [See Geduldig v. Aiello (1974)] (C) is wrong because, as discussed above, legislation excluding abortion-related expenses from government funding has been held not to constitute interference with a woman’s constitutional right to choose to have an abortion. Therefore, the undue burden test does not apply. (D) is incorrect for the same reason as in (C), and also because it does not state the standard that the Court uses to evaluate abortion regulations. Regulations restricting pre-viability abortions will be invalidated if they constitute an “undue burden” on a woman’s right to have an abortion. [Planned Parenthood of Southeastern Pennsylvania v. Casey (1992)]

How well did you know this?
1
Not at all
2
3
4
5
Perfectly
7
Q

A state enacted health care legislation to provide comprehensive insurance coverage on prescription drugs for all of its citizens. The legislation provided state reimbursement for the cost of all prescription drugs with one exception-a drug commonly known as the “abortion pill,” which was prescribed to induce early term abortions without surgery. All other prescription drugs for pregnant women were covered. A pregnant woman who had received a prescription for the drug and was subsequently denied reimbursement filed suit in federal district court challenging the constitutionality of the legislation.
Which of the following best describes the appropriate standard by which the court should review the constitutionality of the state legislation?

A
Because the state legislation does not improperly discriminate against a suspect class or burden a fundamental right, the woman will have to show that the legislation is not rationally related to any legitimate state interest.

B
Because the state legislation discriminates against women by not providing coverage for all of their prescription medications as it does for men, the state will have to demonstrate that the legislation is substantially related to an important government interest.

C
Because the state legislation impinges on a woman’s constitutional right to choose whether to terminate her pregnancy, the state will have to show that the legislation does not constitute an undue burden on that right.

D
Because the state legislation discriminates against women seeking to exercise their fundamental right to terminate their pregnancy in favor of women incurring the regular expenses of pregnancy, the state will have to demonstrate that the legislation is necessary to vindicate a compelling state interest.

A
A
Because the state legislation does not improperly discriminate against a suspect class or burden a fundamental right, the woman will have to show that the legislation is not rationally related to any legitimate state interest.

The court should require the woman to show that the legislation is not rationally related to any legitimate state interest. The Supreme Court has held that the right of privacy includes the right of a woman to have an abortion under certain circumstances without undue interference from the government. However, neither federal nor state government is required to grant medical benefit payments for abortions, even if it grants benefits for childbirth services. The Court has held that a state’s failure to provide funding for a woman’s abortion decision does not constitute interference with her constitutional right to make that decision; hence, such legislation is valid unless the plaintiff can show that it is not rationally related to a legitimate state interest. [See Maher v. Roe (1977)] (B) is incorrect because the legislation does not create a gender-based classification that would require application of an intermediate scrutiny standard. The fact that the restriction applies to a drug prescribed only to women does not establish gender-based discrimination. [See Geduldig v. Aiello (1974)] (C) is wrong because, as discussed above, legislation excluding abortion-related expenses from government funding has been held not to constitute interference with a woman’s constitutional right to choose to have an abortion. Therefore, the undue burden test does not apply. (D) is incorrect for the same reason as in (C), and also because it does not state the standard that the Court uses to evaluate abortion regulations. Regulations restricting pre-viability abortions will be invalidated if they constitute an “undue burden” on a woman’s right to have an abortion. [Planned Parenthood of Southeastern Pennsylvania v. Casey (1992)]

How well did you know this?
1
Not at all
2
3
4
5
Perfectly
8
Q

Under the one person, one vote principle __________.

A
regarding state government districts, almost exact mathematical equality between districts is required

B
regarding state government districts, a variance of more than 3% is invalid

C
regarding congressional districts, a variance of 10% is permissible

D
regarding congressional districts, almost exact mathematical equality between the congressional districts within a state is required

A

D
regarding congressional districts, almost exact mathematical equality between the congressional districts within a state is required

Regarding congressional districts, almost exact mathematical equality between the congressional districts within a state is required under the one person, one vote principle. The rationale is that voting is a fundamental right, diluting one person’s vote compared to another’s raises equal protection concerns, and there is no compelling interest that would justify more than a couple of percentage points difference from district to district.
Regarding state government districts, almost exact mathematical equality between districts is NOT required under the one person, one vote principle. In state government districts, the variance from district to district may not be unjustifiably large. But this is a much more lenient standard than the almost exact mathematical equality standard.
Regarding congressional districts, a variance of 10% is NOT permissible. In congressional districts almost exact mathematical equality is required and a variance of even a couple of percentage points might be ruled invalid.
Regarding state government districts, a variance of more than 3% is NOT invalid under the one person, one vote principle. The variance may not be unjustifiably large, but a variance of even 16% has been found to be valid.

How well did you know this?
1
Not at all
2
3
4
5
Perfectly
9
Q

Under the rules the Supreme Court currently uses to determine whether an abortion regulation is valid, the government may not impose an undue burden on a woman’s ability to obtain an abortion __________.

A
After viability

B
During the first and second trimester

C
During the first trimester

D
Before viability

A

D
Before viability

Under the rules the Supreme Court currently uses to determine whether an abortion regulation is valid, the government may not impose an undue burden on a woman’s ability to obtain an abortion before viability.
The choices involving trimesters are incorrect. In the past, the Supreme Court differentiated its approach to abortion regulation based on the trimester of pregnancy involved, but the Court no longer uses this approach.
After viability, the government may prohibit abortion unless the woman’s health is threatened.

How well did you know this?
1
Not at all
2
3
4
5
Perfectly
10
Q

Which of the following statements is correct regarding government action challenged under the Due Process or Equal Protection Clause where no fundamental right or suspect or quasi-suspect classification is involved?

A
The law will be upheld unless it is arbitrary

B
The law must be the least burdensome means to achieve the legislative goal

C
The burden of proof is on the government to show that the law is necessary

D
The law is valid only if it is substantially related to a legitimate government purpose

A

A
The law will be upheld unless it is arbitrary

If government action is challenged under the Due Process or Equal Protection Clause, and no fundamental right or suspect or quasi-suspect classification is involved, the law will be upheld unless it is arbitrary or irrational. A rational basis standard applies.
The law is valid if it is rationally related to a legitimate government purpose; it need not be substantially related to a legitimate purpose.
The burden of proof is on the CHALLENGER to show that the law is unconstitutional, NOT on the government to show that it is necessary.
The law need NOT be the least burdensome means to achieve the legislative goal. The rational relationship test does not require a tight fit between the goal sought and the law employed-just a rational connection.

How well did you know this?
1
Not at all
2
3
4
5
Perfectly